ML15313A063: Difference between revisions

From kanterella
Jump to navigation Jump to search
(Created page by program invented by StriderTol)
(Created page by program invented by StriderTol)
 
(2 intermediate revisions by the same user not shown)
Line 16: Line 16:


=Text=
=Text=
{{#Wiki_filter:ES-401 Written Examination Review Worksheet CLINTON 2015 Form ES-401-9  
{{#Wiki_filter:ES-401                                     Written Examination Review Worksheet CLINTON 2015                                                         Form ES-401-9
: 1. 2. 3. Psychometric Flaws 4. Job Content Flaws 5. Other 6. 7. 8. Q# LOK LOO (F/H) (1-5) Stem Cues T/F Cred. Partial Job-Minutia #I Back-Q= SRO B/M/N U/E/S Explanation Focus Dist. Link units ward KIA Only 1 L 3 x x y N u The stem states "IRM detectors are withdrawn from the core region during power operation within a tube which extends from the core plate up to the top guide." This question has two problems  
: 1. 2.       3. Psychometric Flaws         4 . Job Content Flaws     5. Other   6. 7.                                     8.
: 1) the IRMs run in dry tubes that extend from the below the reactor vessel to the top guide. So there is no correct answer. 2) the dry tub is inserted into a guide tub . So both the "IRMs are within a dry tube" and "the IRMs are within a guide tube" would be correct.
Q# LOK   LOO (F/H) (1-5) Stem Cues     T/F   Cred. Partial Job- Minutia #I       Back- Q=   SRO B/M/N U/E/S                               Explanation Focus               Dist.       Link           units ward   KIA   Only 1   L     3     x                         x                                 y         N     u   The stem states "IRM detectors are withdrawn from the core region during power operation within a               tube which extends from the core plate up to the top guide." This question has two problems
RESOLUTION
: 1) the IRMs run in dry tubes that extend from the below the reactor vessel to the top guide. So there is no correct answer.
: Question replaced 2 H 3 y B s 3 L 3 y B E The stem now reads ''Which ONE of the following will generate a reactor scram SIGNAL as a result of this loss of power?" Consider changing "SIGNAL* to "input signal".
: 2) the dry tub is inserted into a guide tub . So both the "IRMs are within a dry tube" and "the IRMs are with in a guide tube" would be correct.
The stem conditions would only be a 'h scram? RESOLUTION
RESOLUTION: Question replaced 2   H     3                                                                 y         B     s 3   L     3                                                                 y         B     E   The stem now reads ''Which ONE of the following will generate a reactor scram SIGNAL as a result of this loss of power?" Consider changing "SIGNAL* to "input signal". The stem conditions would only be a 'h scram?
: Change made Instructions (Refer to Section D of ES-401 and Appendix B for additional information regarding each of the following concepts.) 1. Enter the level of knowledge (LOK) of each question as either (F)undamental or (H)igher cognitive level. 2. Enter the level of difficulty (LOO) of each question using a 1 -5 (easy -difficult) rating scale (questions in the 2 -4 range are acceptable)
RESOLUTION: Change made
. 3. Check the appropriate box if a psychometric flaw is identified
 
: The stem lacks sufficient focus to elicit the correct answer (e.g., unclear intent, more information is needed, or too much needless information).
Instructions (Refer to Section D of ES-401 and Appendix B for additional information regarding each of the following concepts.)
The stem or distractors contain cues (i.e., clues. specific determiners
: 1. Enter the level of knowledge (LOK) of each question as either (F)undamental or (H)igher cognitive level.
, phrasing, length, etc). The answer choices are a collection of unrelated true/false statements
: 2. Enter the level of difficulty (LOO) of each question using a 1 - 5 (easy - difficult) rating scale (questions in the 2 - 4 range are acceptable).
. The distractors are not credible; single implausib le distractors should be repaired, more than one is unacceptable
: 3. Check the appropriate box if a psychometric flaw is identified:
. One or more distractors is (are) partially correct (e.g., if the applicant can make unstated assumptions that are not contradicted by stem). 4. Check the appropriate box if a job content error is identified
The stem lacks sufficient focus to elicit the correct answer (e.g ., unclear intent, more information is needed, or too much needless information).
: The question is not linked to the job requirements (i.e., the question has a valid KIA but, as written, is not operational in content). The question requires the recall of knowledge that is too specific for the closed reference test mode (i.e., it is not required to be known from memory).
The stem or distractors contain cues (i.e., clues. specific determiners, phrasing, length, etc).
The question contains data with an unrealistic level of accuracy or inconsistent units (e.g., panel meter in percent with question in gallons). The question requires reverse logic or application compared to the job requirements. 5. Check questions that are sampled for conformance with the approved KIA and those that are designated SRO-only (KIA and license level mismatches are unacceptable)
The answer choices are a collection of unrelated true/false statements.
. 6. Enter question source: (B)ank, (M)odified, or (N)ew. Check that (M)odified questions meet criteria of ES-401 Section 0.2.f. 7. Based on the reviewer's judgment, is the question as written (U)nsatis factory (requiring repair or replacement)
The distractors are not credible; single implausible distractors should be repaired , more than one is unacceptable.
, in need of (E)ditorial enhancement
One or more distractors is (are) partially correct (e.g., if the applicant can make unstated assumptions that are not contradicted by stem).
, or (S)atisfactory?  
: 4. Check the appropriate box if a job content error is identified:
The question is not linked to the job requirements (i.e., the question has a valid KIA but, as written, is not operational in content).
The question requires the recall of knowledge that is too specific for the closed reference test mode (i.e., it is not required to be known from memory).
The question contains data with an unrealistic level of accuracy or inconsistent units (e.g., panel meter in percent with question in gallons).
The question requires reverse logic or application compared to the job requirements.
: 5. Check questions that are sampled fo r conformance with the approved KIA and those that are designated SRO-only (KIA and license level mismatches are unacceptable).
: 6. Enter question source: (B)ank, (M)odified, or (N)ew. Check that (M)odified questions meet criteria of ES-401 Section 0 .2.f.
: 7. Based on the reviewer's judgment, is the question as written (U)nsatisfactory (requiring repair or replacement) , in need of (E)ditorial enhancement, or (S)atisfactory?
: a. At a minimum, explain any "U" ratings (e.g., how the Appendix B psychometric attributes are not being met).
: a. At a minimum, explain any "U" ratings (e.g., how the Appendix B psychometric attributes are not being met).
ES-401 2 Form ES-401-9 1. 2. 3. Psychometric Flaws 4. Job Content Flaws 5. Other 6. 7. 8. Q# LOK LOO (FIH) (1-5) Stem Cues T/F Cred. Partial Job-Minutia #I Back-Q= SRO B/M/N U/E/S Explanation Focus Dist Link units ward KIA Only 4 H 3 y B s 5 H 4 y B s 6 H 4 y N s 7 H 2 y N s 8 L 4 y B s 9 L 3 y N s 10 H 4 y 8 s 11 H 3 y N s 12 H 3 y B s 13 H 2 y N u This questions has 2 problems: 1) If in the condition of the stern the Governor is held in the raise position long enough to see a rise in both watts and current I believe you would also see a rise in vars. Then there would be no correct answer. (on the simulator try raising load from 3000 to 3500 Kw and see if vars change.) 2) JPM "Parallel DG 18 With Offsite Power* will duplicate this question. RESOLUTION: Question revised 14 H 2 y N s 15 H 3 y B s 16 H 4 y N s 17 L 3 y B s 18 H 3 y N s
 
: 1. 2. 3. PsychOmetric Flaws 4. Job Content Flaws 5. Other 6. 7. 8. Q# LOK LOO (FIH) (1-5) Stem Cues T/F Cred Partial Job-Minutia #I Back-Q= SRO B/M/N U/E/S Explanation Focus Dist. Link units ward KJA Only 19 H 3 y B s 20 H 3 y B s 21 H 3 x y N IJ I do not think this is an RO level TS question as written. Consider E changing the bullets to -1 E21-F011 LPCS Pump Min Flow Recirc Valve be closed with fuses removed. -AUTO start of the LPCS disabled. RESOLUTION
ES-401                                                                           2                                                                       Form ES-401-9
: Site supported that the question met the expectations of ROs at Clinton . Editorial changes made. 22 H 3 y B s 23 H 2 y N s 24 L 3 x y B u Between LPRM calibrations APRM gains would be adjusted to account for LPRM detector U235 depletion. RESOLUTIO N: The second correct answer was changed. 25 H 3 y B s 26 L 3 y N s 27 L 3 x y M u If "D" is correct then "a** will be correct. Consider changing "A" and "B" (1) to 'primary but NOT secondary
: 1. 2.       3. Psychometric Flaws         4 . Job Content Flaws     5. Other   6. 7.                                         8.
" . RESOLUTIO N: Requested change made 28 l 3 y B s 29 l 3 y B s 30 L 3 y B s 31 H 3 y N s 32 L 3 y N s 33 L 4 y N s 34 H 3 y N s
Q#   LOK LOO (FIH) (1-5) Stem Cues   T/F   Cred. Partial Job- Minutia #I       Back- Q=   SRO B/M/N U/E/S                                 Explanation Focus               Dist         Link           units ward   KIA Only 4   H     3                                                                 y         B     s 5   H     4                                                                 y         B     s 6   H     4                                                                 y         N     s 7   H     2                                                                 y         N     s 8   L     4                                                                 y         B     s 9     L   3                                                                 y         N     s 10   H     4                                                                 y         8     s 11   H     3                                                                 y         N     s 12   H     3                                                                 y         B     s 13   H     2                                                                 y         N     u   This questions has 2 problems:
: 1. 2. 3. Psychometric Flaws 4. Job Content Flaws 5. Other 6. 7. 8. Q# LOK LOO (F/H) (1-5) Stem Cues T/F Cred. Partial Job-Minutia #I Back-Q= SRO B/M/N U/E/S Explanation Focus Dist. Link units ward KIA Only 35 H 2 y B s 36 L 3 y N s 37 l 3 x y B y "B" prevent boron dilution, *c* prevent injection inside the shroud are E both correct.
: 1) If in the condition of the stern the Governor is held in the raise position long enough to see a rise in both watts and current I believe you would also see a rise in vars. Then there wou ld be no correct answer. (on the simu lator try raising load from 3000 to 3500 Kw and see if vars change .)
See EOP-1A reference provided.
: 2) JPM " Parallel DG 18 With Offsite Power
RESOLUTION:
* will duplicate th is question.
The stem was changed to clarify the question. 38 L 2 y N s 39 H 3 y N s 40 L 3 y 6 s 41 l 3 y B s 42 H 3 y M s 43 H 3 y B s 44 l 3 y N s 45 L 2 y B E List the distracto rs lowest to highest.
RESOLUTION: Question revised 14   H     2                                                                 y         N     s 15     H     3                                                               y         B     s 16     H   4                                                                 y         N     s 17     L     3                                                               y         B     s 18     H     3                                                               y         N     s
i.e. switch "B' and "C". RESOLUTION: Change made 46 H 3 y N s 47 H 3 y N s 48 L 3 y B s 49 H 3 x y B E 1) For the conditions given in the stem is it expected that the cool down rate will increase or decrease with time? *s* will be correct only with an increasing or constant cool down rate. 2) For distracter "A' should it be changed to "Shift RCIR discharge to the reactor vessel"? Because with RCIR in tank to tank vessel level would be unchanged by raising flow. RESOLUTIO N: Stem and distracter changed 50 H 2 y B s
: 1. 2.       3. PsychOmetric Flaws           4. Job Content Flaws   5. Other 6. 7.                                         8.
: 1. 2. 3. Psychometric Flaws 4. Job Content Flaws 5. Other 6. 7. 8. Q# LOK LOO (F/H) (1-5) Stem Cues T/F Cred. Partial Job-Minutia #I Back-Q= SRO B/M/N U/E/S Explanation Focus Dist. Link units ward KIA Only 51 L 3 x y N u It is not clear in the stem if in part (2) of the question we are asking for the action to be taken at the time level is 18ft 9inches (answer D), or at the 15ft 1 inch (Answer C) RESOLUTION
Q# LOK   LOO (FIH) (1-5) Stem Cues   T/F   Cred   Partial Job- Minutia #I Back-   Q= SRO B/M/N U/E/S                                   Explanation Focus             Dist.         Link         units ward KJA Only 19   H     3                                                               y       B   s 20   H     3                                                               y       B   s 21   H     3     x                                                         y       N   IJ I do not think this is an RO level TS question as written. Consider changing the bullets to E
: Question revised. 52 L 3 y B E Change "B" to, "lowest possible driving head and flow from the primary system!:tO reduce the discharge rate." RESOLUTION
                                                                                                    -     1 E21-F011 LPCS Pump Min Flow Recirc Valve be closed with fuses removed .
: Change made 53 L 3 y B s 54 H 3 y M s 55 L 3 y N s 56 H 3 y N s 57 H 4 y N E I think we need to provide the candidates with a time after the loss of offsite power that the plant conditions are current for. (the fact that an SRV is open after a LOFP doesn't=
                                                                                                    -     AUTO start of the LPCS disabled .
ATWS at time 0) Also do you think it is fair to ask an operator to diagnose an A TWS and not give them control rod or APRM info. Maybe we should say you are the BOP and observe the conditions
RESOLUTION: Site supported that the question met the expectations of ROs at Clinton . Editorial changes made.
. RESOLUTION
22   H     3                                                               y       B   s 23   H     2                                                               y       N   s 24     L   3                               x                               y       B   u   Between LPRM calibrations APRM gains would be adjusted to account for LPRM detector U235 depletion.
: Added time = 5 minutes. And justified question based on operator training.
RESOLUTION: The second correct answer was changed.
58 H 3 y M s 59 H 3 y B s 60 H 3 x y B l:J For the conditions of the stem I do not believe you can "maintain E steady states power and balance loop flows" to maintain steady states power for the stem conditions it will require an imbalance in loop flow. RESOLUTION
25   H     3                                                             y       B   s 26     L     3                                                             y       N   s 27     L     3                             x                               y       M   u   If "D" is correct then "a** will be correct. Consider changing "A" and "B" (1) to ' primary but NOT secondary" .
: The conditions of the stem did not require a flow change to comply with Tech Specs, therefore only editorial changes needed to be made 61 L 3 y B s
RESOLUTION: Requested change made 28     l     3                                                             y       B   s 29     l     3                                                             y       B   s 30     L     3                                                             y       B   s 31     H     3                                                             y       N   s 32     L     3                                                             y       N   s 33   L     4                                                             y       N   s 34   H     3                                                             y       N   s
: 1. 2. 3. Psychometric Flaws 4. Job Content Flaws 5. Other 6. 7. 8. Q# LOK LOO (F/H) (1-5) Stem Cues T/F Cred. Partial Job-Minutia #I Back-Q= SRO B/M/N U/E/S Explanation Focus Dist. Link units ward KIA Only 62 L 4 y B s Comment:
: 1. 2.       3. Psychometric Flaws         4. Job Content Flaws     5. Other   6. 7.                                         8.
add an explanation for "D" being incorrect to the answer key. RESOLUTION: Done 63 H 3 y B s 64 L 4 y B s 65 H 4 y B s 66 H 3 y N s 67 L 3 y N s 68 L 3 y B s 69 L 2 y N s 70 L 3 y B s 71 L 2 y B s 72 H 3 y N s 73 H 3 x y B y For a rod drop as described in the stem there would be a ROD s DRIFT Alarm. Therefore "A" would be wrong for the same reason as MC''. RESOLUTION
Q# LOK   LOO (F/H) (1-5) Stem Cues T/F     Cred. Partial Job- Minutia #I     Back- Q=   SRO B/M/N U/E/S                                 Explanation Focus             Dist.         Link         units ward   KIA   Only 35   H     2                                                               y         B     s 36   L     3                                                               y         N     s 37   l     3                             x                                 y         B     y   "B" p revent boron dilution,   *c* prevent injection inside the shroud are both correct. See EOP-1A reference provided.
: Site explained why there would be no ROD DRIFT Alarm. 74 H 4 x y N y CPS 4010.01 step 4.2. 4. Specifically call out Condenser Vacuum E Pumps as needing to be considered for rad conditions. Therefore "C" is correct.
E RESOLUTION: The stem was changed to clarify the quest ion.
Editorial: change Stem to. Under these conditions, when 12erforming the SUBSEQUENT ACTIONs OF CPS 4100.01, what special precaution must be taken to minimize the radiological impact of this event when ... RESOLUTIO N: Site explained why "C" was incorrect.
38   L     2                                                               y         N     s 39   H     3                                                               y         N     s 40   L     3                                                               y         6     s 41   l     3                                                               y         B     s 42   H     3                                                               y         M     s 43   H     3                                                               y         B     s 44   l     3                                                               y         N     s 45   L   2                                                               y         B     E   List the distractors lowest to highest. i.e. switch "B' and "C".
Only editorial changes were needed. 75 L 4 y B s
RESOLUTION: Change made 46   H     3                                                               y         N     s 47   H     3                                                               y         N     s 48   L   3                                                               y         B     s 49   H     3                       x                                       y         B     E         1)   For the conditions given in the stem is it expected that the cool down rate will increase or decrease with time? *s* will be correct only with an increasing or constant cool down rate.
: 1. 2. 3. Psychometric Flaws 4. Job Content Flaws 5. Other 6. 7. 8. Q# LOK LOO (F/H) (1-5) Stem Cues T/F Cred. Partial Job-Minutia #I Back-Q= SRO B/M/N U/E/S Explanation Focus Dist. Link units ward KIA Only s R 0 0 N L y Q u E s T I 0 N s 1 H 3 y y N s 2 H 3 y y N s 3 H 3 y y M s 4 H 3 y y M s 5 H 3 y y N s 6 H 3 x y y N y 1} The question "If the appropriate actions are taken, can a E plant startup commence?"
: 2)   For distracte r "A ' should it be changed to "Shift RCIR discharge to the reactor vessel"? Because with RCIR in tank to tank vessel level would be unchanged by raising flow.
is too vague you can always start up after all appropriate action are taken. Did you mean "Does Tech Spec 3.3.1.1 allow restart without repair of the failed channels?"  
RESOLUTION: Stem and distracter changed 50   H     2                                                               y         B     s
: 2) Could CH A and D have tripped at 19:42: 12:15.!1,? If so could this confuse the candidates
: 1. 2.       3. Psychometric Flaws         4. Job Content Flaws   5. Other 6. 7.                                         8.
. RESOLUTION: Only editorial changes were needed in this question 7 H 3 y N N y This question is written at RO level. i.e. With current conditions s should feedwater be shutoff, should HPCI be shutoff, should MSIVs be closed. RESOLUTION
Q#   LOK LOO (F/H) (1-5) Stem Cues T/F     Cred. Partial Job- Minutia #I Back-   Q= SRO B/M/N U/E/S                                 Explanation Focus             Dist.         Link         units ward KIA Only 51   L     3     x                                                       y       N   u   It is not clear in the stem if in part (2) of the question we are asking for the action to be taken at the time level is 18ft 9inches (answer D),
: Site was able to justify the question as SRO only based on the NRC guide. 8 H 2 y y N s 9 H 3 y N B y This question does not include *selection of procedures
or at the 15ft 1 inch (Answer C)
" only RO E level evaluation of plant condition. Can you change to "enter CPS XXX to spray the containment
RESOLUTION: Question revised.
, etc" RESOLUTION
52   L     3                                                             y       B   E   Change "B" to, "lowest possible driving head and flow from the primary system!:tO reduce the discharge rate."
: Only editorial changes where needed. 10 H 3 y N B s 11 H 3 y y N s
RESOLUTION: Change made 53   L     3                                                             y       B   s 54   H     3                                                             y       M   s 55     L     3                                                             y       N   s 56   H     3                                                             y       N   s 57   H     4                                                             y       N   E   I th ink we need to provide the candidates with a time after the loss of offsite power that the plant conditions are current for. (the fact that an SRV is open after a LOFP doesn't= ATWS at time 0) Also do you think it is fair to ask an operator to diagnose an ATWS and not give them control rod or APRM info. Maybe we should say you are the BOP and observe the conditions.
: 1. 2. 3. Psychometric Flaws 4. Job Content Flaws 5. Other 6. 7. 8. Q# LOK LOO (F/H) (1-5) Stem Cues T/F Cred. Partial Job-Minutia #I Back-Q= SRO B/M/N U/E/S Explanation Focus Dist. Link units ward KIA Only 12 H 4 y y N s 13 H 3 y y N s 14 H 3 y y N s Note: this was accepted as SRO because the candidate was effectively asked to select a section of the SBO procedure to direct. 15 H 3 y y N s 16 H 3 y N N y This question can be answered knowing the power source of the s isolation valves and that the valves should function at 142 F, RO knowledge
RESOLUTION: Added time = 5 minutes. And justified question based on operator training.
. Maybe we could ask "Under these conditions should an emergence event FU1be declared.
58     H     3                                                             y       M   s 59     H     3                                                             y       B   s 60     H     3   x                                                       y       B   l:J For the conditions of the stem I do not believe you can "maintain steady states power and balance loop flows" to maintain steady E
" As the second part of the question. RESOLUTION
states power for the stem conditions it will require an imbalance in loop flow.
: Site was able to justify the question as SRO only based on the NRC guide. 17 L 2 y y N s 18 H 3 y y N s 19 H 4 y y B s 20 H 4 y y N s 21 H 3 y y M s 22 H 3 y y B s 23 L 2 y y N s 24 H 2 x y y B u Why would anyone believe recording a potential LCO would prevent a release? Answers "C & D" RESOLUTION
RESOLUTION: The conditions of the stem did not require a flow change to comply with Tech Specs, therefore only editorial changes needed to be made 61   L     3                                                             y       B   s
: the question was rewritten
: 1. 2.       3. Psychometric Flaws         4. Job Content Flaws     5. Other 6. 7.                                   8.
. 25 L 3 y y N s ES-401 Written Examination Review Worksheet CLINTON 2015 Form ES-401-9  
Q# LOK   LOO (F/H) (1 -5) Stem Cues T/F     Cred. Partial Job- Minutia #I     Back- Q= SRO B/M/N U/E/S                             Explanation Focus             Dist.         Link         units ward   KIA Only 62   L     4                                                                 y       B   s   Comment: add an explanation for "D" being incorrect to the answer key.
: 1. 2. 3. Psychometric Flaws 4. Job Content Flaws 5. Other 6. 7. 8. Q# LOK LOO (F/H) (1-5) Stem Cues T/F Cred. Partial Job-Minutia #I Back-Q= SRO B/M/N U/E/S Explanation Focus Dist. Link units ward KIA Only 1 L 3 x x y N u The stem states "IRM detectors are withdrawn from the core region during power operation within a tube which extends from the core plate up to the top guide." This question has two problems  
RESOLUTION: Done 63   H     3                                                               y       B   s 64   L     4                                                               y       B   s 65   H     4                                                               y       B   s 66   H     3                                                               y       N   s 67     L     3                                                               y       N   s 68     L     3                                                               y       B   s 69     L     2                                                               y       N   s 70     L     3                                                               y       B   s 71     L     2                                                               y       B   s 72     H     3                                                               y       N   s 73     H     3     x                                                         y       B   y   For a rod drop as described in the stem there would be a ROD DRIFT Alarm. Therefore "A" would be wrong for the same reason as MC''.
: 1) the IRMs run in dry tubes that extend from the below the reactor vessel to the top guide. So there is no correct answer. 2) the dry tub is inserted into a guide tub . So both the "IRMs are within a dry tube" and "the IRMs are within a guide tube" would be correct.
RESOLUTION: Site explained why there would be no ROD DRIFT Alarm.
RESOLUTION
74   H     4                             x                                 y       N   y   CPS 401 0.01 step 4.2. 4. Specifically call out Condenser Vacuum Pumps as needing to be considered for rad conditions. Therefore "C" E
: Question replaced 2 H 3 y B s 3 L 3 y B E The stem now reads ''Which ONE of the following will generate a reactor scram SIGNAL as a result of this loss of power?" Consider changing "SIGNAL* to "input signal".
is correct.
The stem conditions would only be a 'h scram? RESOLUTION
Editorial: change Stem to. Under these conditions, when 12erforming the SUBSEQUENT ACTIONs OF CPS 4100.01, what special precaution must be taken to minimize the radiological impact of this event when ...
: Change made Instructions (Refer to Section D of ES-401 and Appendix B for additional information regarding each of the following concepts.) 1. Enter the level of knowledge (LOK) of each question as either (F)undamental or (H)igher cognitive level. 2. Enter the level of difficulty (LOO) of each question using a 1 -5 (easy -difficult) rating scale (questions in the 2 -4 range are acceptable)
RESOLUTION: Site explained why "C" was incorrect. Only editorial changes were needed.
. 3. Check the appropriate box if a psychometric flaw is identified
75   L     4                                                               y       B   s
: The stem lacks sufficient focus to elicit the correct answer (e.g., unclear intent, more information is needed, or too much needless information).
: 1. 2.       3. Psychometric Flaws         4. Job Content Flaws     5. Other 6. 7.                                     8.
The stem or distractors contain cues (i.e., clues. specific determiners
Q# LOK   LOO (F/H) (1 -5) Stem Cues T/F     Cred. Partial Job- Minutia #I     Back- Q = SRO B/M/N U/E/S                                 Explanation Focus             Dist.         Link         units wa rd  KIA Only s   R     0             0   N     L     y             Q     u     E   s     T I     0   N   s 1   H     3                                                               y     y N     s 2   H     3                                                               y     y N     s 3   H     3                                                               y     y M     s 4   H     3                                                               y     y M     s 5   H     3                                                               y     y N     s 6   H     3     x                                                         y     y N     y         1}   The question "If the appropriate actions are taken, can a plant startup commence?" is too vague you can always E
, phrasing, length, etc). The answer choices are a collection of unrelated true/false statements
start up after all appropriate action are taken. Did you mean " Does Tech Spec 3.3.1.1 allow restart without repair of the failed channels?"
. The distractors are not credible; single implausib le distractors should be repaired, more than one is unacceptable
: 2)   Could CH A and D have tripped at 19:42: 12:15.!1,? If so cou ld this confuse the candidates.
. One or more distractors is (are) partially correct (e.g., if the applicant can make unstated assumptions that are not contradicted by stem). 4. Check the appropriate box if a job content error is identified
RESOLUTION: Only editorial changes were needed in this question 7   H     3                                                               y     N N     y   This question is written at RO level. i.e. With current conditions should feedwater be shutoff, should HPCI be shutoff, should MSIVs be closed.
: The question is not linked to the job requirements (i.e., the question has a valid KIA but, as written, is not operational in content). The question requires the recall of knowledge that is too specific for the closed reference test mode (i.e., it is not required to be known from memory).
RESOLUTION: Site was able to justify the question as SRO only based on the NRC guide.
The question contains data with an unrealistic level of accuracy or inconsistent units (e.g., panel meter in percent with question in gallons). The question requires reverse logic or application compared to the job requirements. 5. Check questions that are sampled for conformance with the approved KIA and those that are designated SRO-only (KIA and license level mismatches are unacceptable)
8   H     2                                                               y     y N     s 9   H     3                                                               y     N B     y   This question does not include
. 6. Enter question source: (B)ank, (M)odified, or (N)ew. Check that (M)odified questions meet criteria of ES-401 Section 0.2.f. 7. Based on the reviewer's judgment, is the question as written (U)nsatis factory (requiring repair or replacement)
* selection of procedures" only RO level evaluation of plant condition. Can you change to "enter CPS E
, in need of (E)ditorial enhancement
XXX to spray the containment, etc" RESOLUTION: Only editorial changes where needed.
, or (S)atisfactory?  
10   H     3                                                               y     N B     s 11   H     3                                                               y     y N     s
: 1. 2.       3. Psychometric Flaws         4 . Job Content Flaws   5. Other 6. 7.                                       8.
Q# LOK   LOO (F/H) (1-5) Stem Cues T/F     Cred. Partial Job- Minutia #I Back-     Q= SRO B/M/N U/E/S                               Explanation Focus             Dist.         Link           units ward KIA Only 12   H     4                                                               y     y N   s 13   H     3                                                               y     y N   s 14   H     3                                                               y     y N   s   Note: this was accepted as SRO because the candidate was effectively asked to select a section of the SBO procedure to direct.
15   H     3                                                               y     y N   s 16   H     3                                                               y     N N   y   This question can be answered knowing the power source of the isolation valves and that the valves should function at 142 F, RO knowledge. Maybe we could ask "Under these conditions should an emergence event FU1be declared." As the second part of the question.
RESOLUTION: Site was able to justify the question as SRO only based on the NRC guide.
17   L     2                                                               y     y N   s 18   H     3                                                               y     y N   s 19   H     4                                                               y     y B   s 20   H     4                                                               y     y N   s 21   H     3                                                               y     y M   s 22   H     3                                                               y     y B   s 23   L   2                                                               y     y N   s 24   H     2                       x                                     y     y B   u   Why would anyone believe recording a potential LCO would prevent a release? Answers "C & D" RESOLUTION: the question was rewritten .
25   L   3                                                               y     y N   s
 
ES-401                                     Written Examination Review Worksheet CLINTON 2015                                                         Form ES-401-9
: 1. 2.       3. Psychometric Flaws         4 . Job Content Flaws     5. Other   6. 7.                                     8.
Q# LOK   LOO (F/H) (1-5) Stem Cues     T/F   Cred. Partial Job- Minutia #I       Back- Q=   SRO B/M/N U/E/S                               Explanation Focus               Dist.       Link           units ward   KIA   Only 1   L     3     x                         x                                 y         N     u   The stem states "IRM detectors are withdrawn from the core region during power operation within a               tube which extends from the core plate up to the top guide." This question has two problems
: 1) the IRMs run in dry tubes that extend from the below the reactor vessel to the top guide. So there is no correct answer.
: 2) the dry tub is inserted into a guide tub . So both the "IRMs are within a dry tube" and "the IRMs are with in a guide tube" would be correct.
RESOLUTION: Question replaced 2   H     3                                                                 y         B     s 3   L     3                                                                 y         B     E   The stem now reads ''Which ONE of the following will generate a reactor scram SIGNAL as a result of this loss of power?" Consider changing "SIGNAL* to "input signal". The stem conditions would only be a 'h scram?
RESOLUTION: Change made
 
Instructions (Refer to Section D of ES-401 and Appendix B for additional information regarding each of the following concepts.)
: 1. Enter the level of knowledge (LOK) of each question as either (F)undamental or (H)igher cognitive level.
: 2. Enter the level of difficulty (LOO) of each question using a 1 - 5 (easy - difficult) rating scale (questions in the 2 - 4 range are acceptable).
: 3. Check the appropriate box if a psychometric flaw is identified:
The stem lacks sufficient focus to elicit the correct answer (e.g ., unclear intent, more information is needed, or too much needless information).
The stem or distractors contain cues (i.e., clues. specific determiners, phrasing, length, etc).
The answer choices are a collection of unrelated true/false statements.
The distractors are not credible; single implausible distractors should be repaired , more than one is unacceptable.
One or more distractors is (are) partially correct (e.g., if the applicant can make unstated assumptions that are not contradicted by stem).
: 4. Check the appropriate box if a job content error is identified:
The question is not linked to the job requirements (i.e., the question has a valid KIA but, as written, is not operational in content).
The question requires the recall of knowledge that is too specific for the closed reference test mode (i.e., it is not required to be known from memory).
The question contains data with an unrealistic level of accuracy or inconsistent units (e.g., panel meter in percent with question in gallons).
The question requires reverse logic or application compared to the job requirements.
: 5. Check questions that are sampled fo r conformance with the approved KIA and those that are designated SRO-only (KIA and license level mismatches are unacceptable).
: 6. Enter question source: (B)ank, (M)odified, or (N)ew. Check that (M)odified questions meet criteria of ES-401 Section 0 .2.f.
: 7. Based on the reviewer's judgment, is the question as written (U)nsatisfactory (requiring repair or replacement) , in need of (E)ditorial enhancement, or (S)atisfactory?
: a. At a minimum, explain any "U" ratings (e.g., how the Appendix B psychometric attributes are not being met).
: a. At a minimum, explain any "U" ratings (e.g., how the Appendix B psychometric attributes are not being met).
ES-401 2 Form ES-401-9 1. 2. 3. Psychometric Flaws 4. Job Content Flaws 5. Other 6. 7. 8. Q# LOK LOO (FIH) (1-5) Stem Cues T/F Cred. Partial Job-Minutia #I Back-Q= SRO B/M/N U/E/S Explanation Focus Dist Link units ward KIA Only 4 H 3 y B s 5 H 4 y B s 6 H 4 y N s 7 H 2 y N s 8 L 4 y B s 9 L 3 y N s 10 H 4 y 8 s 11 H 3 y N s 12 H 3 y B s 13 H 2 y N u This questions has 2 problems: 1) If in the condition of the stern the Governor is held in the raise position long enough to see a rise in both watts and current I believe you would also see a rise in vars. Then there would be no correct answer. (on the simulator try raising load from 3000 to 3500 Kw and see if vars change.) 2) JPM "Parallel DG 18 With Offsite Power* will duplicate this question. RESOLUTION: Question revised 14 H 2 y N s 15 H 3 y B s 16 H 4 y N s 17 L 3 y B s 18 H 3 y N s
 
: 1. 2. 3. PsychOmetric Flaws 4. Job Content Flaws 5. Other 6. 7. 8. Q# LOK LOO (FIH) (1-5) Stem Cues T/F Cred Partial Job-Minutia #I Back-Q= SRO B/M/N U/E/S Explanation Focus Dist. Link units ward KJA Only 19 H 3 y B s 20 H 3 y B s 21 H 3 x y N IJ I do not think this is an RO level TS question as written. Consider E changing the bullets to -1 E21-F011 LPCS Pump Min Flow Recirc Valve be closed with fuses removed. -AUTO start of the LPCS disabled. RESOLUTION
ES-401                                                                           2                                                                       Form ES-401-9
: Site supported that the question met the expectations of ROs at Clinton . Editorial changes made. 22 H 3 y B s 23 H 2 y N s 24 L 3 x y B u Between LPRM calibrations APRM gains would be adjusted to account for LPRM detector U235 depletion. RESOLUTIO N: The second correct answer was changed. 25 H 3 y B s 26 L 3 y N s 27 L 3 x y M u If "D" is correct then "a** will be correct. Consider changing "A" and "B" (1) to 'primary but NOT secondary
: 1. 2.       3. Psychometric Flaws         4 . Job Content Flaws     5. Other   6. 7.                                         8.
" . RESOLUTIO N: Requested change made 28 l 3 y B s 29 l 3 y B s 30 L 3 y B s 31 H 3 y N s 32 L 3 y N s 33 L 4 y N s 34 H 3 y N s
Q#   LOK LOO (FIH) (1-5) Stem Cues   T/F   Cred. Partial Job- Minutia #I       Back- Q=   SRO B/M/N U/E/S                                 Explanation Focus               Dist         Link           units ward   KIA Only 4   H     3                                                                 y         B     s 5   H     4                                                                 y         B     s 6   H     4                                                                 y         N     s 7   H     2                                                                 y         N     s 8   L     4                                                                 y         B     s 9     L   3                                                                 y         N     s 10   H     4                                                                 y         8     s 11   H     3                                                                 y         N     s 12   H     3                                                                 y         B     s 13   H     2                                                                 y         N     u   This questions has 2 problems:
: 1. 2. 3. Psychometric Flaws 4. Job Content Flaws 5. Other 6. 7. 8. Q# LOK LOO (F/H) (1-5) Stem Cues T/F Cred. Partial Job-Minutia #I Back-Q= SRO B/M/N U/E/S Explanation Focus Dist. Link units ward KIA Only 35 H 2 y B s 36 L 3 y N s 37 l 3 x y B y "B" prevent boron dilution, *c* prevent injection inside the shroud are E both correct.
: 1) If in the condition of the stern the Governor is held in the raise position long enough to see a rise in both watts and current I believe you would also see a rise in vars. Then there wou ld be no correct answer. (on the simu lator try raising load from 3000 to 3500 Kw and see if vars change .)
See EOP-1A reference provided.
: 2) JPM " Parallel DG 18 With Offsite Power
RESOLUTION:
* will duplicate th is question.
The stem was changed to clarify the question. 38 L 2 y N s 39 H 3 y N s 40 L 3 y 6 s 41 l 3 y B s 42 H 3 y M s 43 H 3 y B s 44 l 3 y N s 45 L 2 y B E List the distracto rs lowest to highest.
RESOLUTION: Question revised 14   H     2                                                                 y         N     s 15     H     3                                                               y         B     s 16     H   4                                                                 y         N     s 17     L     3                                                               y         B     s 18     H     3                                                               y         N     s
i.e. switch "B' and "C". RESOLUTION: Change made 46 H 3 y N s 47 H 3 y N s 48 L 3 y B s 49 H 3 x y B E 1) For the conditions given in the stem is it expected that the cool down rate will increase or decrease with time? *s* will be correct only with an increasing or constant cool down rate. 2) For distracter "A' should it be changed to "Shift RCIR discharge to the reactor vessel"? Because with RCIR in tank to tank vessel level would be unchanged by raising flow. RESOLUTIO N: Stem and distracter changed 50 H 2 y B s
: 1. 2.       3. PsychOmetric Flaws           4. Job Content Flaws   5. Other 6. 7.                                         8.
: 1. 2. 3. Psychometric Flaws 4. Job Content Flaws 5. Other 6. 7. 8. Q# LOK LOO (F/H) (1-5) Stem Cues T/F Cred. Partial Job-Minutia #I Back-Q= SRO B/M/N U/E/S Explanation Focus Dist. Link units ward KIA Only 51 L 3 x y N u It is not clear in the stem if in part (2) of the question we are asking for the action to be taken at the time level is 18ft 9inches (answer D), or at the 15ft 1 inch (Answer C) RESOLUTION
Q# LOK   LOO (FIH) (1-5) Stem Cues   T/F   Cred   Partial Job- Minutia #I Back-   Q= SRO B/M/N U/E/S                                   Explanation Focus             Dist.         Link         units ward KJA Only 19   H     3                                                               y       B   s 20   H     3                                                               y       B   s 21   H     3     x                                                         y       N   IJ I do not think this is an RO level TS question as written. Consider changing the bullets to E
: Question revised. 52 L 3 y B E Change "B" to, "lowest possible driving head and flow from the primary system!:tO reduce the discharge rate." RESOLUTION
                                                                                                    -     1 E21-F011 LPCS Pump Min Flow Recirc Valve be closed with fuses removed .
: Change made 53 L 3 y B s 54 H 3 y M s 55 L 3 y N s 56 H 3 y N s 57 H 4 y N E I think we need to provide the candidates with a time after the loss of offsite power that the plant conditions are current for. (the fact that an SRV is open after a LOFP doesn't=
                                                                                                    -     AUTO start of the LPCS disabled .
ATWS at time 0) Also do you think it is fair to ask an operator to diagnose an A TWS and not give them control rod or APRM info. Maybe we should say you are the BOP and observe the conditions
RESOLUTION: Site supported that the question met the expectations of ROs at Clinton . Editorial changes made.
. RESOLUTION
22   H     3                                                               y       B   s 23   H     2                                                               y       N   s 24     L   3                               x                               y       B   u   Between LPRM calibrations APRM gains would be adjusted to account for LPRM detector U235 depletion.
: Added time = 5 minutes. And justified question based on operator training.
RESOLUTION: The second correct answer was changed.
58 H 3 y M s 59 H 3 y B s 60 H 3 x y B l:J For the conditions of the stem I do not believe you can "maintain E steady states power and balance loop flows" to maintain steady states power for the stem conditions it will require an imbalance in loop flow. RESOLUTION
25   H     3                                                             y       B   s 26     L     3                                                             y       N   s 27     L     3                             x                               y       M   u   If "D" is correct then "a** will be correct. Consider changing "A" and "B" (1) to ' primary but NOT secondary" .
: The conditions of the stem did not require a flow change to comply with Tech Specs, therefore only editorial changes needed to be made 61 L 3 y B s
RESOLUTION: Requested change made 28     l     3                                                             y       B   s 29     l     3                                                             y       B   s 30     L     3                                                             y       B   s 31     H     3                                                             y       N   s 32     L     3                                                             y       N   s 33   L     4                                                             y       N   s 34   H     3                                                             y       N   s
: 1. 2. 3. Psychometric Flaws 4. Job Content Flaws 5. Other 6. 7. 8. Q# LOK LOO (F/H) (1-5) Stem Cues T/F Cred. Partial Job-Minutia #I Back-Q= SRO B/M/N U/E/S Explanation Focus Dist. Link units ward KIA Only 62 L 4 y B s Comment:
: 1. 2.       3. Psychometric Flaws         4. Job Content Flaws     5. Other   6. 7.                                         8.
add an explanation for "D" being incorrect to the answer key. RESOLUTION: Done 63 H 3 y B s 64 L 4 y B s 65 H 4 y B s 66 H 3 y N s 67 L 3 y N s 68 L 3 y B s 69 L 2 y N s 70 L 3 y B s 71 L 2 y B s 72 H 3 y N s 73 H 3 x y B y For a rod drop as described in the stem there would be a ROD s DRIFT Alarm. Therefore "A" would be wrong for the same reason as MC''. RESOLUTION
Q# LOK   LOO (F/H) (1-5) Stem Cues T/F     Cred. Partial Job- Minutia #I     Back- Q=   SRO B/M/N U/E/S                                 Explanation Focus             Dist.         Link         units ward   KIA   Only 35   H     2                                                               y         B     s 36   L     3                                                               y         N     s 37   l     3                             x                                 y         B     y   "B" p revent boron dilution,   *c* prevent injection inside the shroud are both correct. See EOP-1A reference provided.
: Site explained why there would be no ROD DRIFT Alarm. 74 H 4 x y N y CPS 4010.01 step 4.2. 4. Specifically call out Condenser Vacuum E Pumps as needing to be considered for rad conditions. Therefore "C" is correct.
E RESOLUTION: The stem was changed to clarify the quest ion.
Editorial: change Stem to. Under these conditions, when 12erforming the SUBSEQUENT ACTIONs OF CPS 4100.01, what special precaution must be taken to minimize the radiological impact of this event when ... RESOLUTIO N: Site explained why "C" was incorrect.
38   L     2                                                               y         N     s 39   H     3                                                               y         N     s 40   L     3                                                               y         6     s 41   l     3                                                               y         B     s 42   H     3                                                               y         M     s 43   H     3                                                               y         B     s 44   l     3                                                               y         N     s 45   L   2                                                               y         B     E   List the distractors lowest to highest. i.e. switch "B' and "C".
Only editorial changes were needed. 75 L 4 y B s
RESOLUTION: Change made 46   H     3                                                               y         N     s 47   H     3                                                               y         N     s 48   L   3                                                               y         B     s 49   H     3                       x                                       y         B     E         1)   For the conditions given in the stem is it expected that the cool down rate will increase or decrease with time? *s* will be correct only with an increasing or constant cool down rate.
: 1. 2. 3. Psychometric Flaws 4. Job Content Flaws 5. Other 6. 7. 8. Q# LOK LOO (F/H) (1-5) Stem Cues T/F Cred. Partial Job-Minutia #I Back-Q= SRO B/M/N U/E/S Explanation Focus Dist. Link units ward KIA Only s R 0 0 N L y Q u E s T I 0 N s 1 H 3 y y N s 2 H 3 y y N s 3 H 3 y y M s 4 H 3 y y M s 5 H 3 y y N s 6 H 3 x y y N y 1} The question "If the appropriate actions are taken, can a E plant startup commence?"
: 2)   For distracte r "A ' should it be changed to "Shift RCIR discharge to the reactor vessel"? Because with RCIR in tank to tank vessel level would be unchanged by raising flow.
is too vague you can always start up after all appropriate action are taken. Did you mean "Does Tech Spec 3.3.1.1 allow restart without repair of the failed channels?"  
RESOLUTION: Stem and distracter changed 50   H     2                                                               y         B     s
: 2) Could CH A and D have tripped at 19:42: 12:15.!1,? If so could this confuse the candidates
: 1. 2.       3. Psychometric Flaws         4. Job Content Flaws   5. Other 6. 7.                                         8.
. RESOLUTION: Only editorial changes were needed in this question 7 H 3 y N N y This question is written at RO level. i.e. With current conditions s should feedwater be shutoff, should HPCI be shutoff, should MSIVs be closed. RESOLUTION
Q#   LOK LOO (F/H) (1-5) Stem Cues T/F     Cred. Partial Job- Minutia #I Back-   Q= SRO B/M/N U/E/S                                 Explanation Focus             Dist.         Link         units ward KIA Only 51   L     3     x                                                       y       N   u   It is not clear in the stem if in part (2) of the question we are asking for the action to be taken at the time level is 18ft 9inches (answer D),
: Site was able to justify the question as SRO only based on the NRC guide. 8 H 2 y y N s 9 H 3 y N B y This question does not include *selection of procedures
or at the 15ft 1 inch (Answer C)
" only RO E level evaluation of plant condition. Can you change to "enter CPS XXX to spray the containment
RESOLUTION: Question revised.
, etc" RESOLUTION
52   L     3                                                             y       B   E   Change "B" to, "lowest possible driving head and flow from the primary system!:tO reduce the discharge rate."
: Only editorial changes where needed. 10 H 3 y N B s 11 H 3 y y N s
RESOLUTION: Change made 53   L     3                                                             y       B   s 54   H     3                                                             y       M   s 55     L     3                                                             y       N   s 56   H     3                                                             y       N   s 57   H     4                                                             y       N   E   I th ink we need to provide the candidates with a time after the loss of offsite power that the plant conditions are current for. (the fact that an SRV is open after a LOFP doesn't= ATWS at time 0) Also do you think it is fair to ask an operator to diagnose an ATWS and not give them control rod or APRM info. Maybe we should say you are the BOP and observe the conditions.
: 1. 2. 3. Psychometric Flaws 4. Job Content Flaws 5. Other 6. 7. 8. Q# LOK LOO (F/H) (1-5) Stem Cues T/F Cred. Partial Job-Minutia #I Back-Q= SRO B/M/N U/E/S Explanation Focus Dist. Link units ward KIA Only 12 H 4 y y N s 13 H 3 y y N s 14 H 3 y y N s Note: this was accepted as SRO because the candidate was effectively asked to select a section of the SBO procedure to direct. 15 H 3 y y N s 16 H 3 y N N y This question can be answered knowing the power source of the s isolation valves and that the valves should function at 142 F, RO knowledge
RESOLUTION: Added time = 5 minutes. And justified question based on operator training.
. Maybe we could ask "Under these conditions should an emergence event FU1be declared.
58     H     3                                                             y       M   s 59     H     3                                                             y       B   s 60     H     3   x                                                       y       B   l:J For the conditions of the stem I do not believe you can "maintain steady states power and balance loop flows" to maintain steady E
" As the second part of the question. RESOLUTION
states power for the stem conditions it will require an imbalance in loop flow.
: Site was able to justify the question as SRO only based on the NRC guide. 17 L 2 y y N s 18 H 3 y y N s 19 H 4 y y B s 20 H 4 y y N s 21 H 3 y y M s 22 H 3 y y B s 23 L 2 y y N s 24 H 2 x y y B u Why would anyone believe recording a potential LCO would prevent a release? Answers "C & D" RESOLUTION
RESOLUTION: The conditions of the stem did not require a flow change to comply with Tech Specs, therefore only editorial changes needed to be made 61   L     3                                                             y       B   s
: the question was rewritten
: 1. 2.       3. Psychometric Flaws         4. Job Content Flaws     5. Other 6. 7.                                   8.
. 25 L 3 y y N s}}
Q# LOK   LOO (F/H) (1 -5) Stem Cues T/F     Cred. Partial Job- Minutia #I     Back- Q= SRO B/M/N U/E/S                             Explanation Focus             Dist.         Link         units ward   KIA Only 62   L     4                                                                 y       B   s   Comment: add an explanation for "D" being incorrect to the answer key.
RESOLUTION: Done 63   H     3                                                               y       B   s 64   L     4                                                               y       B   s 65   H     4                                                               y       B   s 66   H     3                                                               y       N   s 67     L     3                                                               y       N   s 68     L     3                                                               y       B   s 69     L     2                                                               y       N   s 70     L     3                                                               y       B   s 71     L     2                                                               y       B   s 72     H     3                                                               y       N   s 73     H     3     x                                                         y       B   y   For a rod drop as described in the stem there would be a ROD DRIFT Alarm. Therefore "A" would be wrong for the same reason as MC''.
RESOLUTION: Site explained why there would be no ROD DRIFT Alarm.
74   H     4                             x                                 y       N   y   CPS 401 0.01 step 4.2. 4. Specifically call out Condenser Vacuum Pumps as needing to be considered for rad conditions. Therefore "C" E
is correct.
Editorial: change Stem to. Under these conditions, when 12erforming the SUBSEQUENT ACTIONs OF CPS 4100.01, what special precaution must be taken to minimize the radiological impact of this event when ...
RESOLUTION: Site explained why "C" was incorrect. Only editorial changes were needed.
75   L     4                                                               y       B   s
: 1. 2.       3. Psychometric Flaws         4. Job Content Flaws     5. Other 6. 7.                                     8.
Q# LOK   LOO (F/H) (1 -5) Stem Cues T/F     Cred. Partial Job- Minutia #I     Back- Q = SRO B/M/N U/E/S                                 Explanation Focus             Dist.         Link         units wa rd  KIA Only s   R     0             0   N     L     y             Q     u     E   s     T I     0   N   s 1   H     3                                                               y     y N     s 2   H     3                                                               y     y N     s 3   H     3                                                               y     y M     s 4   H     3                                                               y     y M     s 5   H     3                                                               y     y N     s 6   H     3     x                                                         y     y N     y         1}   The question "If the appropriate actions are taken, can a plant startup commence?" is too vague you can always E
start up after all appropriate action are taken. Did you mean " Does Tech Spec 3.3.1.1 allow restart without repair of the failed channels?"
: 2)   Could CH A and D have tripped at 19:42: 12:15.!1,? If so cou ld this confuse the candidates.
RESOLUTION: Only editorial changes were needed in this question 7   H     3                                                               y     N N     y   This question is written at RO level. i.e. With current conditions should feedwater be shutoff, should HPCI be shutoff, should MSIVs be closed.
RESOLUTION: Site was able to justify the question as SRO only based on the NRC guide.
8   H     2                                                               y     y N     s 9   H     3                                                               y     N B     y   This question does not include
* selection of procedures" only RO level evaluation of plant condition. Can you change to "enter CPS E
XXX to spray the containment, etc" RESOLUTION: Only editorial changes where needed.
10   H     3                                                               y     N B     s 11   H     3                                                               y     y N     s
: 1. 2.       3. Psychometric Flaws         4 . Job Content Flaws   5. Other 6. 7.                                       8.
Q# LOK   LOO (F/H) (1-5) Stem Cues T/F     Cred. Partial Job- Minutia #I Back-     Q= SRO B/M/N U/E/S                               Explanation Focus             Dist.         Link           units ward KIA Only 12   H     4                                                               y     y N   s 13   H     3                                                               y     y N   s 14   H     3                                                               y     y N   s   Note: this was accepted as SRO because the candidate was effectively asked to select a section of the SBO procedure to direct.
15   H     3                                                               y     y N   s 16   H     3                                                               y     N N   y   This question can be answered knowing the power source of the isolation valves and that the valves should function at 142 F, RO knowledge. Maybe we could ask "Under these conditions should an emergence event FU1be declared." As the second part of the question.
RESOLUTION: Site was able to justify the question as SRO only based on the NRC guide.
17   L     2                                                               y     y N   s 18   H     3                                                               y     y N   s 19   H     4                                                               y     y B   s 20   H     4                                                               y     y N   s 21   H     3                                                               y     y M   s 22   H     3                                                               y     y B   s 23   L   2                                                               y     y N   s 24   H     2                       x                                     y     y B   u   Why would anyone believe recording a potential LCO would prevent a release? Answers "C & D" RESOLUTION: the question was rewritten .
25   L   3                                                               y     y N   s}}

Latest revision as of 05:50, 31 October 2019

2015 Clinton Power Station Initial License Examination Form ES-401-9
ML15313A063
Person / Time
Site: Clinton Constellation icon.png
Issue date: 11/06/2015
From:
Division of Reactor Safety III
To:
Zoia, C D
Shared Package
ML15124A054 List:
References
Download: ML15313A063 (9)


Text

ES-401 Written Examination Review Worksheet CLINTON 2015 Form ES-401-9

1. 2. 3. Psychometric Flaws 4 . Job Content Flaws 5. Other 6. 7. 8.

Q# LOK LOO (F/H) (1-5) Stem Cues T/F Cred. Partial Job- Minutia #I Back- Q= SRO B/M/N U/E/S Explanation Focus Dist. Link units ward KIA Only 1 L 3 x x y N u The stem states "IRM detectors are withdrawn from the core region during power operation within a tube which extends from the core plate up to the top guide." This question has two problems

1) the IRMs run in dry tubes that extend from the below the reactor vessel to the top guide. So there is no correct answer.
2) the dry tub is inserted into a guide tub . So both the "IRMs are within a dry tube" and "the IRMs are with in a guide tube" would be correct.

RESOLUTION: Question replaced 2 H 3 y B s 3 L 3 y B E The stem now reads Which ONE of the following will generate a reactor scram SIGNAL as a result of this loss of power?" Consider changing "SIGNAL* to "input signal". The stem conditions would only be a 'h scram?

RESOLUTION: Change made

Instructions (Refer to Section D of ES-401 and Appendix B for additional information regarding each of the following concepts.)

1. Enter the level of knowledge (LOK) of each question as either (F)undamental or (H)igher cognitive level.
2. Enter the level of difficulty (LOO) of each question using a 1 - 5 (easy - difficult) rating scale (questions in the 2 - 4 range are acceptable).
3. Check the appropriate box if a psychometric flaw is identified:

The stem lacks sufficient focus to elicit the correct answer (e.g ., unclear intent, more information is needed, or too much needless information).

The stem or distractors contain cues (i.e., clues. specific determiners, phrasing, length, etc).

The answer choices are a collection of unrelated true/false statements.

The distractors are not credible; single implausible distractors should be repaired , more than one is unacceptable.

One or more distractors is (are) partially correct (e.g., if the applicant can make unstated assumptions that are not contradicted by stem).

4. Check the appropriate box if a job content error is identified:

The question is not linked to the job requirements (i.e., the question has a valid KIA but, as written, is not operational in content).

The question requires the recall of knowledge that is too specific for the closed reference test mode (i.e., it is not required to be known from memory).

The question contains data with an unrealistic level of accuracy or inconsistent units (e.g., panel meter in percent with question in gallons).

The question requires reverse logic or application compared to the job requirements.

5. Check questions that are sampled fo r conformance with the approved KIA and those that are designated SRO-only (KIA and license level mismatches are unacceptable).
6. Enter question source: (B)ank, (M)odified, or (N)ew. Check that (M)odified questions meet criteria of ES-401 Section 0 .2.f.
7. Based on the reviewer's judgment, is the question as written (U)nsatisfactory (requiring repair or replacement) , in need of (E)ditorial enhancement, or (S)atisfactory?
a. At a minimum, explain any "U" ratings (e.g., how the Appendix B psychometric attributes are not being met).

ES-401 2 Form ES-401-9

1. 2. 3. Psychometric Flaws 4 . Job Content Flaws 5. Other 6. 7. 8.

Q# LOK LOO (FIH) (1-5) Stem Cues T/F Cred. Partial Job- Minutia #I Back- Q= SRO B/M/N U/E/S Explanation Focus Dist Link units ward KIA Only 4 H 3 y B s 5 H 4 y B s 6 H 4 y N s 7 H 2 y N s 8 L 4 y B s 9 L 3 y N s 10 H 4 y 8 s 11 H 3 y N s 12 H 3 y B s 13 H 2 y N u This questions has 2 problems:

1) If in the condition of the stern the Governor is held in the raise position long enough to see a rise in both watts and current I believe you would also see a rise in vars. Then there wou ld be no correct answer. (on the simu lator try raising load from 3000 to 3500 Kw and see if vars change .)
2) JPM " Parallel DG 18 With Offsite Power
  • will duplicate th is question.

RESOLUTION: Question revised 14 H 2 y N s 15 H 3 y B s 16 H 4 y N s 17 L 3 y B s 18 H 3 y N s

1. 2. 3. PsychOmetric Flaws 4. Job Content Flaws 5. Other 6. 7. 8.

Q# LOK LOO (FIH) (1-5) Stem Cues T/F Cred Partial Job- Minutia #I Back- Q= SRO B/M/N U/E/S Explanation Focus Dist. Link units ward KJA Only 19 H 3 y B s 20 H 3 y B s 21 H 3 x y N IJ I do not think this is an RO level TS question as written. Consider changing the bullets to E

- 1 E21-F011 LPCS Pump Min Flow Recirc Valve be closed with fuses removed .

- AUTO start of the LPCS disabled .

RESOLUTION: Site supported that the question met the expectations of ROs at Clinton . Editorial changes made.

22 H 3 y B s 23 H 2 y N s 24 L 3 x y B u Between LPRM calibrations APRM gains would be adjusted to account for LPRM detector U235 depletion.

RESOLUTION: The second correct answer was changed.

25 H 3 y B s 26 L 3 y N s 27 L 3 x y M u If "D" is correct then "a** will be correct. Consider changing "A" and "B" (1) to ' primary but NOT secondary" .

RESOLUTION: Requested change made 28 l 3 y B s 29 l 3 y B s 30 L 3 y B s 31 H 3 y N s 32 L 3 y N s 33 L 4 y N s 34 H 3 y N s

1. 2. 3. Psychometric Flaws 4. Job Content Flaws 5. Other 6. 7. 8.

Q# LOK LOO (F/H) (1-5) Stem Cues T/F Cred. Partial Job- Minutia #I Back- Q= SRO B/M/N U/E/S Explanation Focus Dist. Link units ward KIA Only 35 H 2 y B s 36 L 3 y N s 37 l 3 x y B y "B" p revent boron dilution, *c* prevent injection inside the shroud are both correct. See EOP-1A reference provided.

E RESOLUTION: The stem was changed to clarify the quest ion.

38 L 2 y N s 39 H 3 y N s 40 L 3 y 6 s 41 l 3 y B s 42 H 3 y M s 43 H 3 y B s 44 l 3 y N s 45 L 2 y B E List the distractors lowest to highest. i.e. switch "B' and "C".

RESOLUTION: Change made 46 H 3 y N s 47 H 3 y N s 48 L 3 y B s 49 H 3 x y B E 1) For the conditions given in the stem is it expected that the cool down rate will increase or decrease with time? *s* will be correct only with an increasing or constant cool down rate.

2) For distracte r "A ' should it be changed to "Shift RCIR discharge to the reactor vessel"? Because with RCIR in tank to tank vessel level would be unchanged by raising flow.

RESOLUTION: Stem and distracter changed 50 H 2 y B s

1. 2. 3. Psychometric Flaws 4. Job Content Flaws 5. Other 6. 7. 8.

Q# LOK LOO (F/H) (1-5) Stem Cues T/F Cred. Partial Job- Minutia #I Back- Q= SRO B/M/N U/E/S Explanation Focus Dist. Link units ward KIA Only 51 L 3 x y N u It is not clear in the stem if in part (2) of the question we are asking for the action to be taken at the time level is 18ft 9inches (answer D),

or at the 15ft 1 inch (Answer C)

RESOLUTION: Question revised.

52 L 3 y B E Change "B" to, "lowest possible driving head and flow from the primary system!:tO reduce the discharge rate."

RESOLUTION: Change made 53 L 3 y B s 54 H 3 y M s 55 L 3 y N s 56 H 3 y N s 57 H 4 y N E I th ink we need to provide the candidates with a time after the loss of offsite power that the plant conditions are current for. (the fact that an SRV is open after a LOFP doesn't= ATWS at time 0) Also do you think it is fair to ask an operator to diagnose an ATWS and not give them control rod or APRM info. Maybe we should say you are the BOP and observe the conditions.

RESOLUTION: Added time = 5 minutes. And justified question based on operator training.

58 H 3 y M s 59 H 3 y B s 60 H 3 x y B l:J For the conditions of the stem I do not believe you can "maintain steady states power and balance loop flows" to maintain steady E

states power for the stem conditions it will require an imbalance in loop flow.

RESOLUTION: The conditions of the stem did not require a flow change to comply with Tech Specs, therefore only editorial changes needed to be made 61 L 3 y B s

1. 2. 3. Psychometric Flaws 4. Job Content Flaws 5. Other 6. 7. 8.

Q# LOK LOO (F/H) (1 -5) Stem Cues T/F Cred. Partial Job- Minutia #I Back- Q= SRO B/M/N U/E/S Explanation Focus Dist. Link units ward KIA Only 62 L 4 y B s Comment: add an explanation for "D" being incorrect to the answer key.

RESOLUTION: Done 63 H 3 y B s 64 L 4 y B s 65 H 4 y B s 66 H 3 y N s 67 L 3 y N s 68 L 3 y B s 69 L 2 y N s 70 L 3 y B s 71 L 2 y B s 72 H 3 y N s 73 H 3 x y B y For a rod drop as described in the stem there would be a ROD DRIFT Alarm. Therefore "A" would be wrong for the same reason as s MC.

RESOLUTION: Site explained why there would be no ROD DRIFT Alarm.

74 H 4 x y N y CPS 401 0.01 step 4.2. 4. Specifically call out Condenser Vacuum Pumps as needing to be considered for rad conditions. Therefore "C" E

is correct.

Editorial: change Stem to. Under these conditions, when 12erforming the SUBSEQUENT ACTIONs OF CPS 4100.01, what special precaution must be taken to minimize the radiological impact of this event when ...

RESOLUTION: Site explained why "C" was incorrect. Only editorial changes were needed.

75 L 4 y B s

1. 2. 3. Psychometric Flaws 4. Job Content Flaws 5. Other 6. 7. 8.

Q# LOK LOO (F/H) (1 -5) Stem Cues T/F Cred. Partial Job- Minutia #I Back- Q = SRO B/M/N U/E/S Explanation Focus Dist. Link units wa rd KIA Only s R 0 0 N L y Q u E s T I 0 N s 1 H 3 y y N s 2 H 3 y y N s 3 H 3 y y M s 4 H 3 y y M s 5 H 3 y y N s 6 H 3 x y y N y 1} The question "If the appropriate actions are taken, can a plant startup commence?" is too vague you can always E

start up after all appropriate action are taken. Did you mean " Does Tech Spec 3.3.1.1 allow restart without repair of the failed channels?"

2) Could CH A and D have tripped at 19:42: 12:15.!1,? If so cou ld this confuse the candidates.

RESOLUTION: Only editorial changes were needed in this question 7 H 3 y N N y This question is written at RO level. i.e. With current conditions should feedwater be shutoff, should HPCI be shutoff, should MSIVs s be closed.

RESOLUTION: Site was able to justify the question as SRO only based on the NRC guide.

8 H 2 y y N s 9 H 3 y N B y This question does not include

  • selection of procedures" only RO level evaluation of plant condition. Can you change to "enter CPS E

XXX to spray the containment, etc" RESOLUTION: Only editorial changes where needed.

10 H 3 y N B s 11 H 3 y y N s

1. 2. 3. Psychometric Flaws 4 . Job Content Flaws 5. Other 6. 7. 8.

Q# LOK LOO (F/H) (1-5) Stem Cues T/F Cred. Partial Job- Minutia #I Back- Q= SRO B/M/N U/E/S Explanation Focus Dist. Link units ward KIA Only 12 H 4 y y N s 13 H 3 y y N s 14 H 3 y y N s Note: this was accepted as SRO because the candidate was effectively asked to select a section of the SBO procedure to direct.

15 H 3 y y N s 16 H 3 y N N y This question can be answered knowing the power source of the isolation valves and that the valves should function at 142 F, RO s knowledge. Maybe we could ask "Under these conditions should an emergence event FU1be declared." As the second part of the question.

RESOLUTION: Site was able to justify the question as SRO only based on the NRC guide.

17 L 2 y y N s 18 H 3 y y N s 19 H 4 y y B s 20 H 4 y y N s 21 H 3 y y M s 22 H 3 y y B s 23 L 2 y y N s 24 H 2 x y y B u Why would anyone believe recording a potential LCO would prevent a release? Answers "C & D" RESOLUTION: the question was rewritten .

25 L 3 y y N s

ES-401 Written Examination Review Worksheet CLINTON 2015 Form ES-401-9

1. 2. 3. Psychometric Flaws 4 . Job Content Flaws 5. Other 6. 7. 8.

Q# LOK LOO (F/H) (1-5) Stem Cues T/F Cred. Partial Job- Minutia #I Back- Q= SRO B/M/N U/E/S Explanation Focus Dist. Link units ward KIA Only 1 L 3 x x y N u The stem states "IRM detectors are withdrawn from the core region during power operation within a tube which extends from the core plate up to the top guide." This question has two problems

1) the IRMs run in dry tubes that extend from the below the reactor vessel to the top guide. So there is no correct answer.
2) the dry tub is inserted into a guide tub . So both the "IRMs are within a dry tube" and "the IRMs are with in a guide tube" would be correct.

RESOLUTION: Question replaced 2 H 3 y B s 3 L 3 y B E The stem now reads Which ONE of the following will generate a reactor scram SIGNAL as a result of this loss of power?" Consider changing "SIGNAL* to "input signal". The stem conditions would only be a 'h scram?

RESOLUTION: Change made

Instructions (Refer to Section D of ES-401 and Appendix B for additional information regarding each of the following concepts.)

1. Enter the level of knowledge (LOK) of each question as either (F)undamental or (H)igher cognitive level.
2. Enter the level of difficulty (LOO) of each question using a 1 - 5 (easy - difficult) rating scale (questions in the 2 - 4 range are acceptable).
3. Check the appropriate box if a psychometric flaw is identified:

The stem lacks sufficient focus to elicit the correct answer (e.g ., unclear intent, more information is needed, or too much needless information).

The stem or distractors contain cues (i.e., clues. specific determiners, phrasing, length, etc).

The answer choices are a collection of unrelated true/false statements.

The distractors are not credible; single implausible distractors should be repaired , more than one is unacceptable.

One or more distractors is (are) partially correct (e.g., if the applicant can make unstated assumptions that are not contradicted by stem).

4. Check the appropriate box if a job content error is identified:

The question is not linked to the job requirements (i.e., the question has a valid KIA but, as written, is not operational in content).

The question requires the recall of knowledge that is too specific for the closed reference test mode (i.e., it is not required to be known from memory).

The question contains data with an unrealistic level of accuracy or inconsistent units (e.g., panel meter in percent with question in gallons).

The question requires reverse logic or application compared to the job requirements.

5. Check questions that are sampled fo r conformance with the approved KIA and those that are designated SRO-only (KIA and license level mismatches are unacceptable).
6. Enter question source: (B)ank, (M)odified, or (N)ew. Check that (M)odified questions meet criteria of ES-401 Section 0 .2.f.
7. Based on the reviewer's judgment, is the question as written (U)nsatisfactory (requiring repair or replacement) , in need of (E)ditorial enhancement, or (S)atisfactory?
a. At a minimum, explain any "U" ratings (e.g., how the Appendix B psychometric attributes are not being met).

ES-401 2 Form ES-401-9

1. 2. 3. Psychometric Flaws 4 . Job Content Flaws 5. Other 6. 7. 8.

Q# LOK LOO (FIH) (1-5) Stem Cues T/F Cred. Partial Job- Minutia #I Back- Q= SRO B/M/N U/E/S Explanation Focus Dist Link units ward KIA Only 4 H 3 y B s 5 H 4 y B s 6 H 4 y N s 7 H 2 y N s 8 L 4 y B s 9 L 3 y N s 10 H 4 y 8 s 11 H 3 y N s 12 H 3 y B s 13 H 2 y N u This questions has 2 problems:

1) If in the condition of the stern the Governor is held in the raise position long enough to see a rise in both watts and current I believe you would also see a rise in vars. Then there wou ld be no correct answer. (on the simu lator try raising load from 3000 to 3500 Kw and see if vars change .)
2) JPM " Parallel DG 18 With Offsite Power
  • will duplicate th is question.

RESOLUTION: Question revised 14 H 2 y N s 15 H 3 y B s 16 H 4 y N s 17 L 3 y B s 18 H 3 y N s

1. 2. 3. PsychOmetric Flaws 4. Job Content Flaws 5. Other 6. 7. 8.

Q# LOK LOO (FIH) (1-5) Stem Cues T/F Cred Partial Job- Minutia #I Back- Q= SRO B/M/N U/E/S Explanation Focus Dist. Link units ward KJA Only 19 H 3 y B s 20 H 3 y B s 21 H 3 x y N IJ I do not think this is an RO level TS question as written. Consider changing the bullets to E

- 1 E21-F011 LPCS Pump Min Flow Recirc Valve be closed with fuses removed .

- AUTO start of the LPCS disabled .

RESOLUTION: Site supported that the question met the expectations of ROs at Clinton . Editorial changes made.

22 H 3 y B s 23 H 2 y N s 24 L 3 x y B u Between LPRM calibrations APRM gains would be adjusted to account for LPRM detector U235 depletion.

RESOLUTION: The second correct answer was changed.

25 H 3 y B s 26 L 3 y N s 27 L 3 x y M u If "D" is correct then "a** will be correct. Consider changing "A" and "B" (1) to ' primary but NOT secondary" .

RESOLUTION: Requested change made 28 l 3 y B s 29 l 3 y B s 30 L 3 y B s 31 H 3 y N s 32 L 3 y N s 33 L 4 y N s 34 H 3 y N s

1. 2. 3. Psychometric Flaws 4. Job Content Flaws 5. Other 6. 7. 8.

Q# LOK LOO (F/H) (1-5) Stem Cues T/F Cred. Partial Job- Minutia #I Back- Q= SRO B/M/N U/E/S Explanation Focus Dist. Link units ward KIA Only 35 H 2 y B s 36 L 3 y N s 37 l 3 x y B y "B" p revent boron dilution, *c* prevent injection inside the shroud are both correct. See EOP-1A reference provided.

E RESOLUTION: The stem was changed to clarify the quest ion.

38 L 2 y N s 39 H 3 y N s 40 L 3 y 6 s 41 l 3 y B s 42 H 3 y M s 43 H 3 y B s 44 l 3 y N s 45 L 2 y B E List the distractors lowest to highest. i.e. switch "B' and "C".

RESOLUTION: Change made 46 H 3 y N s 47 H 3 y N s 48 L 3 y B s 49 H 3 x y B E 1) For the conditions given in the stem is it expected that the cool down rate will increase or decrease with time? *s* will be correct only with an increasing or constant cool down rate.

2) For distracte r "A ' should it be changed to "Shift RCIR discharge to the reactor vessel"? Because with RCIR in tank to tank vessel level would be unchanged by raising flow.

RESOLUTION: Stem and distracter changed 50 H 2 y B s

1. 2. 3. Psychometric Flaws 4. Job Content Flaws 5. Other 6. 7. 8.

Q# LOK LOO (F/H) (1-5) Stem Cues T/F Cred. Partial Job- Minutia #I Back- Q= SRO B/M/N U/E/S Explanation Focus Dist. Link units ward KIA Only 51 L 3 x y N u It is not clear in the stem if in part (2) of the question we are asking for the action to be taken at the time level is 18ft 9inches (answer D),

or at the 15ft 1 inch (Answer C)

RESOLUTION: Question revised.

52 L 3 y B E Change "B" to, "lowest possible driving head and flow from the primary system!:tO reduce the discharge rate."

RESOLUTION: Change made 53 L 3 y B s 54 H 3 y M s 55 L 3 y N s 56 H 3 y N s 57 H 4 y N E I th ink we need to provide the candidates with a time after the loss of offsite power that the plant conditions are current for. (the fact that an SRV is open after a LOFP doesn't= ATWS at time 0) Also do you think it is fair to ask an operator to diagnose an ATWS and not give them control rod or APRM info. Maybe we should say you are the BOP and observe the conditions.

RESOLUTION: Added time = 5 minutes. And justified question based on operator training.

58 H 3 y M s 59 H 3 y B s 60 H 3 x y B l:J For the conditions of the stem I do not believe you can "maintain steady states power and balance loop flows" to maintain steady E

states power for the stem conditions it will require an imbalance in loop flow.

RESOLUTION: The conditions of the stem did not require a flow change to comply with Tech Specs, therefore only editorial changes needed to be made 61 L 3 y B s

1. 2. 3. Psychometric Flaws 4. Job Content Flaws 5. Other 6. 7. 8.

Q# LOK LOO (F/H) (1 -5) Stem Cues T/F Cred. Partial Job- Minutia #I Back- Q= SRO B/M/N U/E/S Explanation Focus Dist. Link units ward KIA Only 62 L 4 y B s Comment: add an explanation for "D" being incorrect to the answer key.

RESOLUTION: Done 63 H 3 y B s 64 L 4 y B s 65 H 4 y B s 66 H 3 y N s 67 L 3 y N s 68 L 3 y B s 69 L 2 y N s 70 L 3 y B s 71 L 2 y B s 72 H 3 y N s 73 H 3 x y B y For a rod drop as described in the stem there would be a ROD DRIFT Alarm. Therefore "A" would be wrong for the same reason as s MC.

RESOLUTION: Site explained why there would be no ROD DRIFT Alarm.

74 H 4 x y N y CPS 401 0.01 step 4.2. 4. Specifically call out Condenser Vacuum Pumps as needing to be considered for rad conditions. Therefore "C" E

is correct.

Editorial: change Stem to. Under these conditions, when 12erforming the SUBSEQUENT ACTIONs OF CPS 4100.01, what special precaution must be taken to minimize the radiological impact of this event when ...

RESOLUTION: Site explained why "C" was incorrect. Only editorial changes were needed.

75 L 4 y B s

1. 2. 3. Psychometric Flaws 4. Job Content Flaws 5. Other 6. 7. 8.

Q# LOK LOO (F/H) (1 -5) Stem Cues T/F Cred. Partial Job- Minutia #I Back- Q = SRO B/M/N U/E/S Explanation Focus Dist. Link units wa rd KIA Only s R 0 0 N L y Q u E s T I 0 N s 1 H 3 y y N s 2 H 3 y y N s 3 H 3 y y M s 4 H 3 y y M s 5 H 3 y y N s 6 H 3 x y y N y 1} The question "If the appropriate actions are taken, can a plant startup commence?" is too vague you can always E

start up after all appropriate action are taken. Did you mean " Does Tech Spec 3.3.1.1 allow restart without repair of the failed channels?"

2) Could CH A and D have tripped at 19:42: 12:15.!1,? If so cou ld this confuse the candidates.

RESOLUTION: Only editorial changes were needed in this question 7 H 3 y N N y This question is written at RO level. i.e. With current conditions should feedwater be shutoff, should HPCI be shutoff, should MSIVs s be closed.

RESOLUTION: Site was able to justify the question as SRO only based on the NRC guide.

8 H 2 y y N s 9 H 3 y N B y This question does not include

  • selection of procedures" only RO level evaluation of plant condition. Can you change to "enter CPS E

XXX to spray the containment, etc" RESOLUTION: Only editorial changes where needed.

10 H 3 y N B s 11 H 3 y y N s

1. 2. 3. Psychometric Flaws 4 . Job Content Flaws 5. Other 6. 7. 8.

Q# LOK LOO (F/H) (1-5) Stem Cues T/F Cred. Partial Job- Minutia #I Back- Q= SRO B/M/N U/E/S Explanation Focus Dist. Link units ward KIA Only 12 H 4 y y N s 13 H 3 y y N s 14 H 3 y y N s Note: this was accepted as SRO because the candidate was effectively asked to select a section of the SBO procedure to direct.

15 H 3 y y N s 16 H 3 y N N y This question can be answered knowing the power source of the isolation valves and that the valves should function at 142 F, RO s knowledge. Maybe we could ask "Under these conditions should an emergence event FU1be declared." As the second part of the question.

RESOLUTION: Site was able to justify the question as SRO only based on the NRC guide.

17 L 2 y y N s 18 H 3 y y N s 19 H 4 y y B s 20 H 4 y y N s 21 H 3 y y M s 22 H 3 y y B s 23 L 2 y y N s 24 H 2 x y y B u Why would anyone believe recording a potential LCO would prevent a release? Answers "C & D" RESOLUTION: the question was rewritten .

25 L 3 y y N s